MyPatentBar Old Answers Flashcards
- Which of the following statements is true regarding a product-by-process claim?
(A) Product-by-process claims cannot vary in scope from each other.
(B) Product-by-process claims may only be used in chemical cases.
(C) A lesser burden of proof may be required to make out a case of prima facie obviousness for product-by-process claims than is required to make out a prima facie case of obviousness when the product is claimed in the conventional fashion.
(D) It is proper to use product-by-process claims only when the process is patentable.
(E) It is proper to use product-by-process claims only when the product is incapable of description in the conventional fashion.
- ANSWER: (C). MPEP § 2113, page 2100-51, citing In re Fessman, 489 F.2d 742, 744, 180 USPQ 324, 326 (CCPA 1974). (A) and (E) are incorrect because “[t]he fact that it is necessary for an applicant to describe his product in product-by-process terms does not prevent him from presenting claims of varying scope.” MPEP § 2173.05(p), item (I). (D) is incorrect even if it is not necessary to describe the product in product-by-process form. (D) is incorrect because “determination of patentability is based on the product itself. The patentability of a product does not depend on its method of production. If the product in the product-by-process claim is the same as or obvious from a product of the prior art, the claim is unpatentable even thought the prior art was made by a different process.” MPEP § 2113, [p.2100-51]. (B) is incorrect. “A claim to a device, apparatus, claim, or composition of matter may contain a reference to the process in which it is intended to be used…so long as it is clear that the claim is directed to the product and not the process.” MPEP § 2173.05(p), item (I) [p.2100-174].
- A non-final Office action contains, among other things, a restriction requirement between
two groups of claims, (Group 1 and Group 2). Which of the following, if included in a timely
reply under 37 C.F.R. § 1.111, preserves applicant’s right to petition the Commissioner to review
the restriction requirement?
I. Applicant’s entire reply to the restriction requirement is: “The examiner erred in
distinguishing between Group 1 and Group 2, and therefore the restriction
requirement is respectfully traversed and no election is being made, in order that
applicant’s right to petition the Commissioner to review the restriction
requirement is preserved.”
II. Applicant’s entire reply to the restriction requirement is: “Applicant elects
Group 1 and respectfully traverses the restriction requirement, because the
examiner erred in requiring a restriction between Group 1 and Group 2.”
III. Applicant’s reply distinctly points out detailed reasons why applicant believes the
examiner erred in requiring a restriction between Group 1 and Group 2, and
additionally sets forth, “Applicant therefore respectfully traverses the restriction
requirement and no election is being made, in order that applicant’s right to
petition the Commissioner to review the restriction requirement is preserved.”
IV. Applicant’s reply distinctly points out detailed reasons why applicant believes the
examiner erred in requiring a restriction between Group 1 and Group 2, and
additionally sets forth, “Applicant therefore respectfully traverses the restriction
requirement and elects Group 2.
(A) I.
(B) II.
(C) III.
(D) IV.
(E) None of the above.
ANSWER: (D). 37 C.F.R. § 1.111(b); MPEP §§ 818.03(a)-(c). (I) is incorrect since the
traversal does not distinctly point out the supposed errors in the examiner’s action, and no
election is made. 37 C.F.R. § 1.143. (II) is incorrect since the traversal does not distinctly point
out the supposed errors in the examiner’s action. (III) is incorrect since no election is made. (E)
is incorrect because (D) is correct.
**10. Mario Lepieux was a member of a Canadian national hockey team touring Europe. While traveling through Germany (a WTO member country) in December 1998, Mario conceived of an aerodynamic design for a hockey helmet that offered players improved protection while reducing air resistance during skating. Upon Mario’s return to Canada (a NAFTA country), he enlisted his brothers Luigi and Pepe Lepieux to help him market the product under the tradename “Wing Cap.” On February 1, 1999, without Mario’s knowledge or permission, Luigi anonymously published a promotional article written by Mario and fully disclosing how the Wing Cap was made and used. The promotional article was published in Moose Jaw Monthly, a regional Canadian magazine that is not distributed in the United States. The Wing Cap was first reduced to practice on March 17, 1999. A United States patent application properly naming Mario as the sole inventor was filed September 17, 1999. That application has now been rejected as being anticipated by the Moose Jaw Monthly article.
- Which of the following statements is most correct?
(A) The promotional article cannot be used as prior art because the Wing Cap had not been reduced to practice at the time the article appeared in the regional Canadian magazine.
(B) The regional Canadian magazine article is not prima facie prior art because it was published without Mario’s knowledge or permission.
(C) The regional Canadian magazine article is not prima facie prior art because it appeared in a regional Canadian publication and does not evidence knowledge or use in the United States.
(D) The promotional article in the regional Canadian magazine constituted an offer to sell that operates as an absolute bar against Mario’s patent application.
(E) Mario, as the inventor, can overcome the rejection by filing an affidavit under 37 C.F.R. § 1.132 establishing that he is the inventor, and the article describes his work.
- Which of the following statements is most correct?
(A) In a priority contest against another inventor, Mario can rely on his activities in Canada in establishing a date of invention.
(B) In a priority contest against another inventor, Mario can rely on his activities in Germany in establishing a date of invention.
(C) Mario can rely on his activities in Canada in establishing a date of invention prior to publication of the regional Canadian magazine article.
(D) (A) and (C).
(E) (A), (B), and (C).
- ANSWER: (E). MPEP § 716.10. There is no requirement that a publication describe something that has actually been reduced to practice before the publication can act as a prior art reference. Thus, statement (A) is not correct. With regard to statement (B), there is no requirement under 35 U.S.C. § 102 that a publication be made with an inventor’s knowledge or permission before it constitutes prior art. Statement (C) is incorrect at least because the Wing Cap was “described in a printed publication in…a foreign country” (35 U.S.C. § 102(a)) before Mario’s filing date and is therefore presumptive prior art. (D) is incorrect because even if the promotional article constituted an offer to sell, it was not in this country and was made less than a year prior to Mario’s filing date. 35 U.S.C. § 102(b).
- ANSWER: (E). Mario may rely on activities in both Germany (a WTO member country) and Canada (a NAFTA country) in establishing a date of invention prior to publication of the Moose Jaw Monthly article or in establishing priority. 35 U.S.C. § 104; see also MPEP § 715.01(c).
- On August 7, 1997, practitioner Costello filed a patent application identifying Laurel,
Abbot, and Hardy as inventors. Each named inventor assigned his patent rights to Burns just
prior to the application being filed. Laurel and Abbot, alone, jointly invented the subject matter
of independent claim 1 in the application. Hardy contributed to inventing the subject matter of
claim 2. Claim 2 properly depends upon claim 1. The examiner rejected claim 1 and claim 2
under 35 U.S.C. § 102(a) as anticipated by a journal article by Allen, dated July 9, 1997. Laurel,
Abbot, and Hardy are readily available to provide evidence in support of and sign an antedating
affidavit under 37 C.F.R. § 1.131 showing reduction to practice of the subject matter of claims 1
and 2 prior to July 9, 1997. Which of the following may properly make an affidavit under 37
C.F.R. § 1.131 to overcome the rejection of claims 1 and 2.?
(A) Laurel and Abbot.
(B) Laurel, Abbot, and Hardy.
(C) Laurel, Hardy and Burns.
(D) Burns only.
(E) None of the above.
- ANSWER: (B). MPEP 715.04. (A) is incorrect since it cannot be shown that less than all
the inventors invented the subject matter of claim 2. (C) and (D) are incorrect since the assignee
can make an affidavit under 37 C.F.R. § 1.131, only when it is not possible to produce the
affidavit of the inventor. The facts indicate that all inventors were readily available produce the
affidavit. (E) is incorrect since (B) is correct.
**17. Smith invented a laminate. In a patent application, Smith most broadly disclosed the laminate as comprising a transparent protective layer in continuous, direct contact with a light-sensitive layer without any intermediate layer between the transparent protective layer and the light-sensitive layer. The prior art published two years before the effective filing date of Smith’s application included a laminate containing a transparent protective layer and a light-sensitive layer held together by an intermediate adhesive layer. Which of the following is a proper claim that would overcome a 35 U.S.C. § 102 rejection based on the prior art?
(A) 1. A laminate comprising a transparent protective layer and a light-sensitive layer.
(B) 1. A laminate comprising a transparent protective layer and a light-sensitive layer
which is in continuous and direct contact with the transparent protective layer.
(C) 1. A laminate comprising a transparent protective layer and a light-sensitive layer,
but not including an adhesive layer.
(D) (A) and (B).
(E) (B) and (C).
ANSWER: (E) is correct because (B) and (C) are correct. (A) does not overcome the prior art because the broad “comprising” language permits the laminate to include additional layers, such as an adhesive layer. MPEP 2111.03. (B) overcomes a 35 U.S.C. § 102 rejection because the claim requires a light-sensitive layer to be in continuous and direct contact with the transparent protective layer, whereas the prior art interposes an adhesive layer between the light-sensitive layer and transparent protective layer. (C) also avoids the prior art by using a negative limitation to particularly point out and distinctly claim that Smith does not claim any laminate including an adhesive layer. MPEP 2173.05(i).
A recent test taker noted that the question currently asked on teh Prometric exam is a variant. The USPTO added a claim limitation distinction to correct the answer choice. The new question has two choices between using “Comprising” OR “Consisting of”. The correct answer is the same answer from April 00 am practice exam.
ALSO - Got a variant on 3/30/13. Facts were basically the same (used the word “contiguous”), but the question asked which would be rejected as anticipated?
(A) 1. A laminate comprising a transparent protective layer and a light-sensitive layer.
(B) 1. A laminate comprising a transparent protective layer and a light-sensitive layer which is in contiguous and direct contact with the transparent protective layer.
(C) 1. A laminate comprising a transparent protective layer and a light-sensitive layer, but not including an adhesive layer.
(D) 1. A laminate consisting of a transparent protective layer and a light-sensitive layer
(E) 1. A laminate consisting of a transparent protective layer and a light-sensitive layer which is in continuous and direct contact with the transparent protective layer.
Answer was A.
**Questions #23 and #24 from the April 2000 (AM) patent bar exam are in the Prometric database.
Please answer questions 23 and 24 based on the following facts. Jo invented a new and unobvious technique for inexpensively manufacturing a chemical that has been used in paper mills for years to bleach paper. Tommie developed a new and unobvious technique to clean-up toxic waste spills. Jo and Tommie collaborated to invent a method to clean-up toxic waste spills using the chemical made according to the unobvious technique invented by Jo. The inventions have been assigned to your client Dowel Chemical Company. You prepared a single patent application fully disclosing and claiming each invention. Claims 1-9 were directed to the method of manufacturing; claims 10-19 were directed to the method of cleaning up toxic waste spills; and claim 20 was directed to a method of cleaning up toxic waste spills using the chemical manufactured in accordance with claim 1. Both inventors approve the application, but Tommie is unavailable to sign an oath before an upcoming statutory bar. Accordingly, you are instructed to immediately file the application without an executed oath. On June 1, 1999, you file the application along with an information sheet to identify the application. However, you do not notice that Tommie was inadvertently left off the list of inventors on the information sheet, which listed Jo as the sole inventor. After receiving a Notice to File Missing Parts, you submit an oath executed by both Jo and Tommie. No paper was filed to change the named inventive entity. You later receive an Office action requiring restriction between Jo’s invention and Tommie’s invention. In reply to the restriction requirement, you elect Jo’s invention, cancel claims 10-20, and immediately file a divisional application directed to the invention of claims 10-19. Claim 20 was omitted from the divisional application. The divisional application includes a specific reference to the original application and is files with an inventor’s oath executed by Tommie only. The divisional application incorporated the original application by reference.
- Which of the following statements is correct?
(A) Because the original application as filed only named Jo as an inventor, Tommie’s divisional application is not entitled to the filing date of the original Application because there is no common inventor between the original application and the divisional application.
(B) The incorrect inventorship listed on the information sheet of the original application was never properly corrected and, therefore, any patent issuing on that application will be invalid under 35 U.S.C. § 116 unless the inventorship is later corrected.
(C) After canceling claims 10-20, it is necessary to change the named inventive entity in the original application by filing a petition including a statement identifying Tommie as being deleted and acknowledging that Tommie’s invention is no longer being claimed in the application and an appropriate fee.
(D) Written consent of the Dowel Chemical Company is required before Tommie can be deleted as an inventor in the original application.
(E) It is necessary in the divisional application to file a petition including a statement identifying Jo as being deleted as an inventor and acknowledging that Jo’s invention is not being claimed in the divisional application.
- Which of the following statements is most correct?
(A) Since claim 20 was omitted from the divisional application as filed, it cannot be added to the divisional application by subsequent Amendment because such an Amendment would constitute new matter.
(B) It was improper to include Tommie and Jo as joint inventors in the patent application.
(C) The examiner may properly make a provisional obviousness-type double patenting rejection in the divisional application based on the parent application, but that rejection may be readily overcome with the filing of a terminal disclaimer.
(D) Because the inventive entity of the amended parent application is different than the inventive entity of the divisional application, the examiner may reject the claims of the divisional application under the provisions of 30 U.S.C. § 102(e).
(E) Statements (A), (B), (C) and (D) are each incorrect.
- ANSWER: (C). The original mistake in omitting Tommie from the list of inventors was automatically corrected by filing the oath executed by both Jo and Tommie. 37 C.F.R. § 1.48(f)(1). Under 37 C.F.R. § 148(b), a change to the inventive entity is thereafter required upon cancellation of the non-elected claims. (B) is wrong because inventorship was automatically corrected with the filing of the corrected oath, (A) is wrong because Tommie was properly named as a co-inventor in the parent application, and 35 U.S>C. § 120 requires, inter alia, only one common inventor. (D) is incorrect because an assignee’s written consent is not required if an inventor is being deleted because the prosecution of the application results in the cancellation of claims so that fewer than a;; of the currently named inventors are the actual inventors of the inventions being claimed in the application. 37 C.F.R. §148(b). (E) is incorrect because the divisional application never named Jo as an inventor so there is no need to correct the inventorship.
- ANSWER: (E). Statement (A) is incorrect at least because the parent application, which included claim 20, was incorporated by reference. Therefore, adding claim 20 does not constitute the addition of new matter. Statement (B) is wrong because at least claim 20 is properly viewed as a joint invention (“JO and Tommie collaborated to invent a method to clean-up toxic waste spills using the chemical made according to the unobvious technique invented by Jo.”) MPEP §§ 605.07, item (E), and 2137.01. Under 35 U.S.C. § 116, inventors map apply for a patent jointly even if they did not make the same type or amount of contribution and did not each make a contribution to the subject matter of every claim in the patent. Statement (C) is wrong because 35 U.S.C. § 121 precludes such a rejection. Statement (D) is wrong at least because Tommie is entitled to the filing date of the parent application.
**27. On February 8, 1999, you filed a patent application that you prepared for Mr. Bond. The application contains only one claim. The application disclosed a composition having 20%A, 20%B, and either 60%C or 60%D. Claim 1 is as follows:
Claim 1. A composition useful for bonding semiconductor materials to metals, comprising 20%A, 20%B, and 60%C.”
The examiner found a patent to Gold, dated March 8, 1998, which only disclosed and claimed a composition, having 20%A, 20%B, and 60%C, and also taught that the composition would only be useful for insulating metals from corrosion. The examiner rejected Claim 1 under 35 U.S.C. § 102(a) as anticipated by Gold, in an Office action dated August 9, 1999. Which of the following is most likely to overcome the rejection, and comports with proper PTO rules and procedure?
(A) Filing a reply, on March 9, 2000, with a petition for a three-month extension and the fee for a three-month extension, traversing the rejection on the ground that Gold does not disclose using the composition for bonding semiconductor materials to metals, and therefore does not disclose all the elements of Claim 1.
(B) Filing a reply, on September 9, 1999, traversing the rejection on the ground that Gold does not disclose using the composition for bonding semi-conductor materials to metals, and therefore does not disclose all the elements of Claim 1.
(C) Filing a reply on October 9, 1999, amending Claim 1 to state as follows: “Claim 1. A composition comprising: 20%A, 20%B, and 60%D.” In the reply, pointing out why the amendment gives the claim patentable novelty.
(D) Filing a reply on October 9, 1999, traversing the rejection on the grounds that the patent to Gold teaches away from using the invention in the manner taught in Bond’s application.
(E) Filing (i) a 37 C.F.R. § 1.132 affidavit objectively demonstrating the commercial success of the invention as claimed, and (ii) a reply containing an argument why the claimed invention is patentable, but no amendment to Claim 1.
ANSWER: (C). MPEP § 2111.02. (A), (B), and (D) are incorrect since the “use” recited in the preamble in Claim 1 does not result in a structural difference between the claimed invention and the disclosure in the Gold patent. In re Casey, 370 F.2d 576 (CCPA 1967). (A) is further incorrect since the reply would not be filed within the statutory period. (D) is further incorrect since the rejection is not under 35 U.S.C. § 103, and any “teaching away” in the Gold patent is not applicable to the rejection under 35 U.S.C. § 102(a). (E) is incorrect since evidence of commercial success, relevant to secondary considerations concerning rejections under 35 U.S.C. § 103, is not relevant to overcoming rejections under 35 U.S.C. § 102(a). (C) is correct since the amendment is timely filed, supported by the disclosure, and renders the rejection under 35 U.S.C. § 102(a) inapplicable.
- Nonobviousness of a claimed invention may be demonstrated by:
(A) producing evidence that all the beneficial results are expected based on the
teachings of the prior art references.
(B) producing evidence of the absence of a property the claimed invention would be
expected to possess based on the teachings of the prior art.
(C) producing evidence showing that unexpected results occur over less than the
entire claimed range.
(D) producing evidence showing that the unexpected properties of a claimed invention
have a significance less than equal to the expected properties.
(E) (A), (B), (C) and (D).
- ANSWER: (B). See Ex parte Mead Johnson & Co., 227 USPQ 78 (Bd. Pat. App. & Int.
1985); MPEP 716.02(a) page 700-155 (Absence of Expected Property is Evidence of
Nonobviousness). (A) is incorrect. “Expected beneficial results are evidence of obviousness of
the claimed invention.” In re Gershon, 372 F.2d 535, 538, 152 USPQ 602, 604 (CCPA 1967),
MPEP 716.02(c). (C) is incorrect. Unexpected results must be commensurate in scope with the
claimed invention. In re Clemens, 622 F.2d 1029, 1036, 206 USPQ 298, 296 (CCPA 1980);
MPEP 716.02(d). (D) is incorrect. Evidence not showing that the unexpected properties of a
claimed invention have a significance equal to or greater than the expected properties may be
insufficient to rebut the evidence of obviousness. In re Nolan, 553 F.2d 1261, 1267, 193 USPQ
641, 645 (CCPA 1977); MPEP 716.02(c). (E) is incorrect because (A), (C) and (D) are
incorrect.
- You have just received an Office action rejecting all of your claims in your patent
application as anticipated under 35 U.S.C. § 102(a) using published declassified material as the
reference. The examiner explains that the declassified material is being used as prima facie
evidence of prior knowledge as of the printing date. The published declassified material contains
information showing that it was printed six months before the filing date of the application, and
that it was published two months after the application’s filing date. You correctly note that
although the printing date precedes your application filing date by six months, you note that the
publication was classified as of its printing date (thus, available only for limited distribution even
when the application was filed), and was not declassified until its publication date (when it
became available to the general public). Each element of the claimed invention is described in
the publication of the declassified material. Which of the following statements is true?
(A) The rejection is not supported by the reference.
(B) The publication is not available as a reference because it did not become available
to the general public until after the filing date of your patent application.
(C) The publication is prima facie evidence of prior knowledge even though it was
available only for limited distribution as of its printing date.
(D) The publication constitutes an absolute statutory bar.
(E) It is not possible to use a Rule 131 affidavit or declaration to antedate the printing
date of the publication.
- ANSWER: (C). As stated in MPEP § 707.05(f), “For the purpose of anticipation
predicated upon prior knowledge under 35 U.S.C. 102(a), the above noted declassified material
may be taken as prima facie evidence of such prior knowledge as of its printing date even though
such material was classified at that time.” (A) is incorrect. The reference supports the rejection
inasmuch as each element of the claimed invention is disclosed in the reference. (B), (D), and
(E) are not the most correct. MPEP § 707.05(f).
**36. A petition to make a patent application special may be filed without fee in which of the following cases?
(A) The petition is supported by applicant’s birth certificate showing applicant’s age is 62.
(B) The petition is supported by applicant’s unverified statement that applicant’s age is 65.
(C) The petition is supported by applicant’s statement that there is an infringing device actually on the market, that a rigid comparison of the alleged infringing device with the claims of the application has been made, and that applicant has made a careful and thorough search of the prior art.
(D) The petition is accompanied by a statement under 37 C.F.R. § 1.102 by applicant explaining the relationship of the invention to safety of research in the field of recombinant DNA research.
(E) The petition is accompanied by applicant’s statement explaining how the invention contributes to the diagnosis, treatment or prevention of HIV/AIDS or cancer.
ANSWER: (B). (A) is wrong because MPEP § 708.02, IV, recites, “An application may be made special upon filing a petition including any evidence showing that the applicant is 65 years of age, or more, such as a birth certificate or applicant’s statement. No fee is required with such a petition.” (C), (D), and (E) are wrong because a fee is required with respect to each petition. MPEP § 708.02, II, VII, and X, respectively.
**42. Which of the following can never properly be available as prior art for purposes of a rejection under 35 U.S.C. § 102(a)?
(A) A drawing, labeled “Prior Art,” submitted by the applicant.
(B) Canceled matter in an application that matured into a U.S. patent where the matter is not published in the patent.
(C) An abandoned patent application referenced in a publication available to the public.
(D) The combination of two references, where one of the references is used merely to explain the meaning of a term used in the primary reference.
(E) A reference authored only by applicant, and published less than one year prior to the effective filing date of applicant’s patent application.
ANSWER: (E) is the correct answer. (A) is incorrect since admissions, including figures labeled “prior art” may be used. MPEP 2129. (B) is incorrect since canceled matter in the application file of a U.S. patent becomes available as prior art as of the date the application issues into a patent. See MPEP 2127, and Ex parte Stalego, 154 USPQ 52, 53 (Bd. App. 1966) cited therein. (C) is incorrect since an abandoned patent application may become evidence of prior art when it has been appropriately disclosed, as, for example, when it is referenced in a publication. See 37 C.F.R. § 1.14(a)(3)(iv); MPEP 2127; and Lee Pharmaceutical v. Kreps, 577 F.2d 610, 613, 198 USPQ 601, 605 (9th Cir. 1978) cited in MPEP 2127. (D) is incorrect because multiple reference rejections under 35 U.S.C. § 102 may be used where one reference is used to merely explain a term used in the primary reference. See MPEP 2131.01 and In re Baxter Travenol Labs., 952 F.2d 388, 21 USPQ2d 1281 (Fed. Cir. 1991) cited therein. (E) is correct since the reference is not by “another.”
- Which of the following, if any, is true?
(A) The loser in an interference in the PTO is estopped from later claiming he or she
was the first to invent in a Federal District Court since the loser must win in the
PTO or he/she will lose the right to contest priority.
(B) A person being sued for infringement may file a request for reexamination
without first obtaining the permission of the Court in which the litigation is taking
place.
(C) A practitioner may not represent spouses, family members or relatives before the
PTO since such representation inherently creates a conflict of interest and a
practitioner is likely to engage in favoritism over his/her other clients.
(D) Employees of the PTO may not apply for a patent during the period of their
employment and for two years thereafter.
(E) None of the above.
- ANSWER: (B). Any person at any time may file a request for reexamination. 35 U.S.C. §
- As to (A) loser may appeal to District Court under 35 U.S.C. § 146. As to (C), there is no
prohibition regarding spouses, family members, and other relatives. As to (D) according to 35
U.S.C. § 4, employees are prohibited during the period of their employment and one year
thereafter. As to (E), (B) is true.
- Which of the following is true?
(A) A final decision by a United States District Court finding a patent to be invalid
will have no binding effect during reexamination since the PTO may still find the
claims of the patent to be valid.
(B) A final decision by a United States District Court finding a patent to be valid will
have no binding effect during reexamination since the PTO may still find the
claims of the patent to be invalid.
(C) Once the Court of Appeals for the Federal Circuit determines that the claims of a
patent are valid, the USPTO may not find such claims invalid based upon newly
discovered art.
(D) If a patentee fails to disclose prior art to the PTO during regular prosecution, the
only way that a patentee can disclose later discovered prior art to the PTO after
issuance is by filing a request for reexamination.
(E) Once a patent claim is found valid during a District Court Proceeding then the
patent claims are entitled to a higher standard of patentability and the presumption
of validity can only be rebutted by clear and convincing evidence in a concurrent
or later reexamination proceeding.
- ANSWER: (B) is the most correct answer. As to (A) and (B) see MPEP 2286, page 2200-
97, stating, “[t]he issuance of a final district court decision upholding validity during a
reexamination also will have no binding effect on the examination of the reexamination.” Thus,
(A) is incorrect because a final holding of invalidity is binding on the PTO. As to (C), the PTO
may discover new art and find claims unpatentable as that art would raise a substantial new
question. MPEP §§ 2216 and 2286. As to (D), the patentee could file a prior art statement under
35 U.S.C. § 301, or disclose prior art in reissue application if the original patent (through error
without deceptive intent) is defective or claims more or less than should be claimed. As to (E),
preponderance of evidence standard does not change in reexamination proceedings. MPEP 2286.
- Mr. Roberts, an American citizen touring a vineyard, saw a unique grape-squeezing
machine in France. The machine was highly efficient, and produced excellent wine. The
vineyard owner was not hiding the machine. It was out of public view and was the only one of
its kind. The vineyard owner had built it himself several years earlier, and no drawing or
technical description of the machine was ever made. The vineyard made only local sales of its
wines. Using his photographic memory, Roberts went back to his hotel and made technical
drawings of what he had seen. Upon his return to the United States, Roberts promptly prepared
and filed a patent application directed to the machine. Which of the following statements is
correct?
(A) Roberts may not obtain a patent on the machine because it was known by others
before Mr. Roberts made technical drawings of the machine.
(B) Roberts may not obtain a patent on the machine because wine made by the
machine had been sold more than a year before Roberts’ application filing date.
(C) Roberts is entitled to a patent because a goal of the patent system is public
disclosure of technical advances, and the machine would not have been disclosed
to the public without Roberts’ efforts.
(D) Roberts may not obtain a patent on the machine because the vineyard owner was
not hiding the machine and therefore the machine was in public use more than a
year before Roberts’ application filing date.
(E) Statements (A), (B), (C) and (D) are each incorrect.
- ANSWER: (E). Roberts is not entitled to a patent because he did not himself invent the
subject matter sought to be patented. 35 U.S.C. § 102(f). Therefore, statement (C) cannot be
correct. Statement (A) is incorrect because, although the machine was known by others, it was
not known by others in this country as required under 35 U.S.C. § 102(a). Similarly, statements
(B) and (D) are incorrect because, even if there was a sale or public use more than a year before
Roberts’ filing date, it was not “in this country” as required by 35 U.S.C. § 102(b).
- Which of the following may not be properly used as prior art for purposes of rejecting a
claim under 35 U.S.C. § 102(b) in an application having an effective filing date of Monday,
May 3, 1999?
(A) A journal article, published Saturday, May 2, 1998, disclosing all the claimed
elements and fully teaching how to make and use the invention as claimed.
(B) A foreign patent, published March 3, 1998, which applicant referenced in the
application when claiming foreign priority based on the foreign application date,
and applicant submitted a certified copy of the original foreign application.
(C) Applicant’s statement in a declaration under 37 C.F.R. § 1.132 that although the
invention as claimed had been offered for sale in department stores in New York
during 1997, this was done only to analyze consumer acceptance of the packaging
in which the invention is marketed.
(D) A journal article, published May 1, 1997, disclosing all the elements of the claim
and teaching how to make and use the claimed invention. The examiner used the
article in combination with another journal article in a previous non-final Office
action to reject the same claim under 35 U.S.C. § 103.
(E) All of the above.
- ANSWER: (A). MPEP 706.02(a). (B) is incorrect since the foreign patent, published more
than one year before the effective filing date of the application, would serve as a bar regardless
of the attempt to claim priority. 35 U.S.C. § 102(b). (C) is incorrect since market testing is not a
proper exception to a statutory bar under 35 U.S.C. § 102(b). MPEP 2133.03(e)(6). Also, the
facts do not involve testing the invention, but only the packaging. (D) is incorrect since a
reference may be used to reject claims under both 35 U.S.C. § 102 and 35 U.S.C. § 103. MPEP
2141.01. (E) is incorrect since only (A) may not properly be used.
(VARIENT):
Another variant of 102(b)question:
French App 1-4-03 turns to French patent 12-5-03 then US app in 4-1-04. which 102 may examiner use to reject prior art?
a. 102a
b. 102b
c. 102d
d. 102a & 102d
e. 102b & 102d
**44. Which of the following statements best correctly describes current PTO practice and
procedure?
(A) Where a patent discloses subject matter being claimed in an application
undergoing examination, if the patent’s designation of inventorship differs from
that of the application, then the patent’s designation of inventorship does not raise
a presumption of inventorship regarding the subject matter disclosed but not
claimed in the patent so as to justify a rejection under 35 U.S.C. § 102(f).
(B) The fact that a claim recites various components, all of which can be
argumentatively assumed to be old, provides a proper basis for a rejection under
35 U.S.C. § 102(f).
(C) A person can be an inventor without having contributed to the conception of the
invention.
(D) In arriving at conception, an inventor may not consider and adopt ideas and
materials derived from other sources such as an employee or hired consultant.
(E) It is essential for the inventor to be personally involved in reducing the invention
to actual practice.
- ANSWER: (A) is the most correct answer. See MPEP § 2137, p.2100-89. (B) is incorrect.
The mere fact that the claim recites components, all of which can be argumentatively assumed to
be old, does not provide a basis for rejection under 35 U.S.C. § 102(f). Ex parte Billottet, 192
USPQ 413, 415 (Bd. App. 1976); MPEP§ 2137. (C) is incorrect. One must contribute to the
conception to be an inventor. In re Hardee, 223 USPQ 1122, 1123 (Comm’r Pat. 1984). Unless
a person contributes to the conception of the invention, the person cannot be an inventor. Fiers
v. Revel, 984 F.2d 1164, 1168, 25 USPQ2d 1601, 1604-05 (Fed. Cir. 1993); MPEP § 2137.01
(section styled “An Inventor Must Contribute To The Conception of the Invention”). (D) is
incorrect. An inventor may consider and adopt suggestions from many sources. Morse v.
Porter, 155 USPQ 280, 283 (Bd. Pat. Inter. 1965); New England Braiding Co. v. A.W. Cheterton
Co., 970 F.2d. 878, 883, 23 USPQ2d 1622, 1626 (Fed. Cir. 1992); MPEP § 2137.01 (section
styled “As Long As The Inventor Maintains Intellectual Domination Over Making The
Invention, Ideas, Suggestions, And Materials May Be Adopted From Others”). (E) is incorrect.
In re DeBaun, 687 F.2d 459, 463, 214 USPQ 933, 936 (CCPA 1982) (“there is no requirement
that the inventor be the one to reduce the invention to practice so long as the reduction to practice
was done on his behalf”); MPEP § 2137.01 (section styled “The Inventor Is Not Required To
Reduce The Invention To Practice).
**2. On December 31, 1998, Sam Practitioner files a notice of appeal in a patent application
assigned to ABC Corp. after the examiner has rejected all of the claims on prior art. Within two
months he sends in his appeal brief and three months after the examiner’s answer is filed the case
is sent to the Board of Patent Appeals and Interferences (Board). Subsequently, while reading
the Official Gazette Sam notices that a patent issued to XYZ Corp. on October 26, 1999,
contains claims which read on an unclaimed embodiment in the ABC application, which is an
invention that is not within the scope of the invention claimed in the ABC application. The ABC
application was filed one month after the issuance of the XYZ patent. Upon learning of the XYZ
patent, ABC Corp. wants to provoke an interference by adding additional claims to its
application relating to the previously unclaimed embodiment. It is October 18, 2000 and Sam
comes to you for advice. Which of the following is the best and correct course of action?
(A) Since the ABC application is at the Board of Patent Appeals and Interferences
already, Sam need only request that the case be transferred to the Interference part
of the Board where an interference can be declared between the ABC application
and the XYZ patent.
(B) Sam should file an amendment adding the claims copied from the XYZ patent and
the Board is required to enter the amendment.
(C) Sam should promptly file an amendment containing the claims copied from the
XYZ patent and request entry. If the Board declines to enter the amendment, Sam
should file a separate, continuation application no later than October 26, 2000,
containing the claims copied from the XYZ patent as well as claims previously
appealed, and then, to avoid the rendering of a decision of the Board, he should
promptly inform the clerk of the Board in writing that they have decided to refile
and abandon the application containing an appeal waiting a decision.
(D) Sam should file an amendment containing the claims copied from the XYZ patent
and ask that the interference between the ABC application and the XYZ patent be
considered while the case is at the Board.
(E) Sam should wait until the appeal is decided before filing an amendment to
incorporate claims copied from the XYZ patent and to provoke an interference.
There is no benefit to filing new claims since only allowable claims will be
considered during an interference.
- ANSWER: (C). 35 U.S.C. § 135(b) requires that the claim be made in the ABC patent
within one year of the issuance of the XYZ patent. MPEP § 1211.01 states that there is no
obligation resting on the Board to consider new or amended claims submitted while the case is
on appeal. MPEP § 1210 states that when an application is refiled, the Board should be promptly
notified. Failure to notify the Board may result in the Board’s refusing an otherwise proper
request to vacate its decision. See also MPEP § 2307.03. (E) is incorrect as 35 U.S.C. § 135(b)
requires the claim to be made within one year. (D) is incorrect because the Board may refuse the
amendment and because the claims have not yet been determined to be allowable. Note that the
XYZ patent was filed before the ABC application and is therefore prior art under 35 U.S.C. §
102(e). See MPEP § 2306 and 2307.02. (A) is incorrect because the claims are not in the
application, no interference could be declared and such a “transfer” is not feasible. (B) is
incorrect because the Board is not required to enter the amendment and Sam may forfeit his
opportunity to present the claims within one year if he does not act promptly (as in answer (C)).
- Which of the following statements regarding a proper prior art reference is true?
(A) Canceled matter in the application file of a U.S. patent is a prior art reference as
of the filing date under 35 U.S.C. 102(e).
(B) Where a patent refers to and relies on the disclosure of a copending subsequently
abandoned application, such disclosure is not available as a reference.
(C) Where the reference patent claims the benefit of an earlier filed, copending but
subsequently abandoned application which discloses subject matter in common
with the patent, and the abandoned application has an enabling disclosure for the
common subject matter and the claimed matter in the reference patent, the
effective date of the reference patent as to the common subject matter is the filing
date of the reference patent.
(D) Matter canceled from the application file wrapper of a U.S. patent may be used as
prior art as of the patent date.
(E) All foreign patents are available as prior art as of the date they are translated into
English.
- ANSWER: (D). 35 U.S.C. § 102(a). As explained in MPEP § 901.01, the “matter canceled
from the application file wrapper of a U.S. patent may be used as prior art as of the patent date in
that it then constitutes prior public knowledge under 35 U.S.C. 102(a), In re Lund,376 F.2d 982,
153 USPQ 625 (CCPA 1967). See also MPEP § § 2127 and § 2136.02.” (A) is incorrect. 35
U.S.C. § 102(e). As stated in MPEP § 901.01, “Canceled matter in the application file of a U.S.
patent is not a proper reference as of the filing date under 35 U.S.C. 102(e), see Ex parte Stalego,
154 USPQ 52, 53 (Bd. App. 1966).” (B) is incorrect. As stated in MPEP § 901.02, “In re
Heritage, 182 F.2d 639, 86 USPQ 160 (CCPA 1950), holds that where a patent refers to and
relies on the disclosure of a copending abandoned application, such disclosure is available as a
reference. See also In re Lund, 376 F.2d 982, 153 USPQ 625 (CCPA 1967).” (C) is incorrect.
As MPEP § 901.02 indicates, where the reference patent claims the benefit of a copending but
abandoned application which discloses subject matter in common with the patent, and the
abandoned application has an enabling disclosure of the common subject matter and claimed
matter in the reference patent, the effective date of the reference as to the common subject matter
is the filing date of the abandoned application. In re Switzer, 77 USPQ 1, 612 O.G. 11 (CCPA
1948); Ex parte Peterson, 63 USPQ 99 (Bd. App. 1944); and Ex parte Clifford,49 USPQ 152
(Bd. App. 1940).” (E) is incorrect. As stated in MPEP § 901.05, “In general, a foreign patent,
the contents of its application, or segments of its content should not be cited as a reference until
its date of patenting or publication can be confirmed by an examiner’s review of a copy of the
document.”
- Which of the following is not a USPTO recommendation or requirement?
(A) Claims should be arranged in order of scope so that the first claim presented is the least restrictive.
(B) Product and process claims should be separately grouped.
(C) Every application should contain no more than three dependent claims.
(D) A claim which depends from a dependent claim should not be separated from that dependent claim by any claim which does not also depend from the dependent claim.
(E) Each claim should start with a capital letter and end with a period.
- ANSWER: (C). The USPTO does not require or recommend a minimum or maximum number of dependent claims. 37 C.F.R. § 1.75(c). (A) is a USPTO recommendation. See MPEP 608.01(m) (“Claims should preferably be arranged in order of scope so that the first claim presented is the least restrictive.”). (B) is a USPTO recommendation. See MPEP 608.01(m) (“Similarly, product and process claims should be separately grouped.”). (D) is a PTO recommendation. See MPEP 608.01(n), part IV. (E) is a USPTO requirement. See MPEP 608.01(m) (“Each claim begins with a capital letter and ends with a period.”).
- In June 1998, Jack and Jill, a married couple, are vacationing in Vietnam (not a WTO
country) when they encounter a man selling bamboo knives for cleaning fish. The particular
curvature of the bamboo both lends support to the knife to prevent it from bending and breaking
and facilitates cleaning inside the fish. Jill takes a picture of Jack with the knife cleaning the
fish. Subsequently, in November 1998, when Jack returns to the United States he begins to make
and sell an identical knife to the one seen in Vietnam. In July 1999, he files a patent application
claiming the nearly identical knife. Jack discloses no prior art during the prosecution of his
application and fails to mention the knife he saw in Vietnam. The examiner finds no prior art
similar to the claimed knife, and Jack is awarded a patent in December 2000. Meanwhile, Jill
divorces Jack, and associates with Sam. Unfortunately, Sam is penniless. To raise cash, Sam
and Jill begin selling a knife identical to the one Jack produces, only Sam and Jill make their
knife out of plastic. The knives of Sam and Jill sell like hotcakes. Jack sues for infringement.
Jill and Sam come to you for advice. Which of the following is not true?
(A) Jack is entitled to patent protection since Vietnam is not a WTO country and
evidence of the Vietnamese knife cannot be used against him to reject his patent
claims.
(B) Jack had a duty under 37 C.F.R. §1.56 to disclose his discovery of the bamboo
knife in Vietnam to the examiner during the original patent prosecution.
(C) Since the use in Vietnam was not in this country, it does not constitute a public
use bar under 35 U.S.C. § 102(b).
(D) If Jill’s attorney files a request for reexamination, it will be denied because the
picture is not a patent or printed publication.
(E) Although Jack marketed the invention before obtaining a patent, the patent claims
cannot be invalidated under 35 U.S.C. § 102(a) since Jack’s making and selling of
the knife cannot be used against him under 35 U.S.C. § 102(a).
- ANSWER: (A) is the most correct answer. Answer (A) is not true since Jack did not invent
the knife, therefore he is not entitled to a patent. Jack derived the invention from another, and
the picture of Jack with the Vietnamese knife is evidence of derivation. 35 U.S.C. § 102(f);
MPEP § 2137. Answer (B) is correct in that Jack should have disclosed “all information
material to patentability,” including the existence of the Vietnamese knife, during the original
patent prosecution. (C) is correct in that to qualify as prior under 35 U.S.C. § 102(b), the use
must be in this country. (D) is correct in that a request for reexamination must be based upon
patents and printed publications. (E) is correct in that public use derived from the inventor’s
own work cannot be used against the inventor under 35 U.S.C. § 102(a). MPEP § 2132.
- You have agreed to represent an independent inventor in connection with a patent
application that was filed in the USPTO by the inventor on a pro se basis. As filed, the
application included a detailed written description that, when viewed together with four
accompanying color photographs, enabled one of ordinary skill in the pertinent art to make and
use the invention. The application was filed with an inventor’s declaration in compliance with
37 CFR 1.63, a small entity statement (independent inventor) under 37 CFR 1.27, and all
necessary small entity filing fees. MEGACORP, a very large multi- national corporation,
licensed rights in the invention after the application was filed.
You have been asked to suggest steps to remove any formal objections that can be expected from
the patent examiner, without incurring unnecessary government fees. You determine that the
first color photograph is the only practical medium by which to disclose certain aspects of the
claimed invention, but that the substance of the remaining photographs could readily be
illustrated through ordinary ink drawings. You correctly recall that the Office announced in the
Official Gazette in May 2001 (1246 OG 106) that it is sua sponte waiving 37 CFR 1.84(a)(2)(iii),
and is no longer requiring a black and white photocopy of any color drawing or color
photograph. Which of the following represents the most reasonable advice to the independent
inventor?
(A) Submit a request for approval of drawing changes wherein the first photograph is
labeled “Figure 1” and the remaining photographs are cancelled in favor of
corresponding ink drawings labeled Figures 2 through 4; and immediately
withdraw the claim for small entity status because of the license to MEGACORP.
(B) Submit a request for approval of drawing changes wherein the first photograph is
labeled “Figure 1” and the remaining photographs are cancelled in favor of
corresponding ink drawings labeled Figures 2 through 4; and submit a petition for
acceptance of Figure 1 in the form of a color photograph along with three sets of
the color photograph, a proposed amendment to insert language concerning the
color photograph as the first paragraph of the specification and the required
petition fee. The photographs must be sufficient quality that all details in the
photographs are reproducible in a printed patent.
(C) Submit a request for approval of drawing changes wherein the first photograph is
labeled “Figure 1” and the remaining photographs are cancelled in favor of
corresponding ink drawings labeled Figures 2 through 4.
(D) Immediately withdraw the claim for small entity status because of the license to
MEGACORP and submit to the USPTO the difference between the small entity
filing fee and the large entity filing fee.
(E) File a rewritten application as a continuation application including a color
photograph as Figure 1, ink drawings as Figures 2-4, a new inventor’s declaration
and a small entity filing fee.
- ANSWER: The most correct answer is (B). 37 CFR 1.84(a)(2), MPEP § 608.02; Notice
(Interim Waiver of Parts of 37 CFR 1.84 and 1.165, and Delay in the Enforcement of the Change
in 37 CFR 1.84(e) to No Longer Permit Mounting of Photographs) in Official Gazette May 22,
2001, 1246 OG 106 (“In summary, the USPTO has sua sponte waived 37 CFR 1.84(a)(2)(iii) and
1.165(b) and is no longer requiring a black and white photocopy of any color drawing or
photograph”). (A) is wrong because a petition under 37 CFR 1.84 is required to avoid an
objection to the color photographs. Also, since small entity status was properly established at the
time of filing, the inventor is entitled to maintain small entity status until any issue fee is due. 37
CFR 1.27(g)(1). (C) – (E) are also wrong because they do not provide for the required petition
under 37 CFR 1.84. In (D), the change in small entity status after the application was filed does
not require the inventor to retroactively pay a large entity filing fee. Additionally, (E) is wrong
because the inventor would be required to file a large entity filing fee and a continuation
application therefore does not achieve the stated goal of avoiding unnecessary government fees.
**6. In connection with the utility of an invention described in a patent application, which of
the following conforms to proper USPTO practice and procedure?
(A) A deficiency under 35 U.S.C. § 101 also creates a deficiency under 35 U.S.C.
§ 112, first paragraph.
(B) To overcome a rejection under 35 U.S.C. § 101, it must be shown that the claimed
device is capable of achieving a useful result on all occasions and under all
conditions.
(C) A claimed invention is properly rejected under 35 U.S.C. § 101 as lacking utility
if the particular embodiment disclosed in the patent lacks perfection or performs
crudely.
(D) To overcome a rejection under 35 U.S.C. § 101, it is essential to show that the
claimed invention accomplishes all its intended functions.
(E) A claimed invention lacks utility if it is not commercially successful.
- ANSWER: The most correct answer is (A). As stated in MPEP § 2107.01 (IV). A
deficiency under 35 U.S.C. § 101 also creates a deficiency under 35 U.S.C. § 112, first
paragraph. See In re Brana, 51 F.3d 1560, 34 USPQ2d 1436 (Fed. Cir. 1995); In re Jolles, 628
F.2d 1322, 1326 n.10, 206 USPQ 885, 889 n.11 (CCPA 1980); In re Fouche, 439 F.2d 1237,
1243, 169 USPQ 429, 434 (CCPA 1971) (“If such compositions are in fact useless, appellant’s
specification cannot have taught how to use them.”). (B) is not correct. MPEP § 2107 (II), and
see Brooktree Corp. v. Advanced Micro Devices, Inc., 977 F.2d 1555, 1571, 24 USPQ2d 1401,
1412 (Fed. Cir. 1992); and E.I. du Pont De Nemours and Co. v. Berkley and Co., 620 F.2d 1247,
1260 n.17, 205 USPQ 1, 10 n.17 (8th Cir. 1980). (C), (D) and (E) are not correct. MPEP § 2107
(II), and see E.I. du Pont De Nemours and Co. v. Berkley and Co., 620 F.2d 1247, 1260 n.17,
205 USPQ 1, 10 n.17 (8th Cir. 1980).
- In which of the following final Office action rejections is the finality of the Office action
rejection proper?
(A) The final Office action rejection is in a second Office action and uses newly cited
art under 35 U.S.C. § 102(b) to reject unamended claims that were objected to but
not rejected in a first Office action.
(B) The final Office action rejection is in a first Office action in a continuation- in-part
application where at least one claim includes subject matter not present in the
parent application.
(C) The final Office action rejection is in a first Office action in a continuing
application, all claims are drawn to the same inve ntion claimed in the parent
application, and the claims would have been properly finally rejected on the
grounds and art of record in the next Office action if they had been entered in the
parent application.
(D) The final Office action rejection is in a first Office action in a substitute
application that contains material that was presented after final rejection in an
earlier application but was denied entry because the issue of new matter was
raised.
(E) None of the above.
- ANSWER: (C) is the correct answer. MPEP § 706.07(b). (A) is incorrect because a final
rejection is not proper on a second action if it includes a rejection on newly cited art other than
information submitted in an information disclo sure statement under 37 CFR 1.97(c). MPEP
§ 706.07(a). (B) is incorrect because it is improper to make final a first Office action in a
continuation- in-part application where any claim includes subject matter not present in the parent
application. MPEP § 706.07(b). (D) is incorrect because it is improper to make final a first
Office action in a substitute application where that application contains material, which was
presented in the earlier application after final rejection, or closing of prosecution but was denied
entry because the issue of new matter was raised. MPEP § 706.07(b). (E) is incorrect because
(C) is correct.
- Which of the following is not in accord with proper USPTO practice and procedure?
(A) A written description as filed in a nonprovisional patent application is presumed
adequate under 35 U.S.C. § 112 in the absence of evidence or reasoning to the
contrary.
(B) An examiner may show that a written description as filed in a nonprovisional
patent application is not adequate by presenting a preponderance of evidence why
a person of ordinary skill in the art would not recognize in the applicant’s
disclosure a description of the invention defined by the claims.
(C) A general allegation of “unpredictability in the art” is sufficient to support a
rejection of a claim for lack of an adequate written description.
(D) When filing an amendment, a practitioner should show support in the original
disclosure for new or amended claims.
(E) When there is substantial variation within a genus, an applicant must describe a
sufficient variety of species to reflect the variation within the genus. - Sam is a sole proprietor of Sam’s Labs, which has no other employees. Sam invented a
new drug while doing research under a Government contract. Sam desires to file a patent
application for his invention and assign it to Sam’s Labs. Sam has licensed Rick, also a sole
proprietor with no employees, to make and use his invention. Sam wants to claim small entity
status when filing a patent application for his invention. Sam also wants to grant the
Government a license, but will not do so if he will be denied small entity status. Sam has limited
resources and wants to know whether, how, and to what extent he may claim small entity status.
Which of the following is not accurate with respect to proper USPTO procedure in relation to
applications filed on or after January 1, 2001?
(A) Sam’s Labs is a small business concern for the purposes of claiming small entity
status for fee reduction purposes.
(B) If Sam grants a license to the Government resulting from a rights determination
under Executive Order 10096, it will not constitute a license so as to prohibit
claiming small entity status.
(C) The establishment of small entity status permits the recipient to pay reduced fees
for all patent application processing fees charged by the USPTO.
(D) Sam may establish small entity status by a written assertion of entitlement to
small entity status. A written assertion must: (i) be clearly identifiable; (ii) be
signed; and (iii) convey the concept of entitlement to small entity status, such as
by stating that applicant is a small entity, or that small entity status is entitled to
be asserted for the application or patent.
(E) While no specific words or wording are required to assert small entity status, the
intent to assert small entity status must be clearly indicated in order to comply
with the assertion requirement.
- ANSWER: (C), not being in accord with proper USPTO practice and procedure, is the most
correct answer. As stated in “Guidelines for Examination of Patent Applications under 35
U.S.C. 112, ¶ 1, ‘Written Description’ Requirement,” 66 F.R. 1099, 1107 (Jan. 5, 2001), middle
column, “A general allegation of ‘unpredictability in the art’ is not a sufficient reason to support
a rejection for lack of adequate written description”; MPEP § 2163, paragraph III. A. (pg. 2100-
166) (8th Ed.). (A), being in accord with proper USPTO practice and procedure, is not correct.
As stated in “Guidelines for Examination of Patent Applications under 35 U.S.C. 112, ¶ 1,
‘Written Description’ Requirement,” 66 F.R. 1099, 1107 (Jan. 5, 2001), left column, “A
description as filed is presumed to be adequate…” MPEP § 2163, paragraph III. A. (pg. 2100-
166) (8th Ed.). (B), being in accord with proper USPTO practice and procedure, is not correct.
As stated in “Guidelines for Examination of Patent Applications under 35 U.S.C. 112, ¶ 1,
‘Written Description’ Requirement,” 66 F.R. 1099, 1107 (Jan. 5, 2001), “A description as filed is
presumed to be adequate, unless or until sufficient evidence or reasoning to the contrary has been
presented by the examiner to rebut the presumption.65 …The examiner has the initial burden of
presenting by a preponderance of evidence why a person skilled in the art would not recognize in
an applicant’s disclosure a description of the invention defined by the claims.66” (footnotes not
reproduced); MPEP § 2163, paragraph III. A. (pg. 2100-166) (8th Ed.). (D), being in accord with
proper USPTO practice and procedure, is not correct. As stated in “Guidelines for Examination
of Patent Applications under 35 U.S.C. 112, ¶ 1, ‘Written Description’ Requirement,” 66 F.R.
1099, 1107 (Jan. 5, 2001), left column, “[W]hen filing an amendment, applicant should show
support in the original disclosure for new or amended claims.59” Footnote 59 states, “See MPEP
§§ 714.02 and 2163.06 (‘Applicant should…specifically point out the support for any
amendments made to the disclosure.’)”; MPEP § 2163, paragraph III. A. 3. (b) (pg. 2100-165)
(8th Ed.). (E), being in accord with proper USPTO practice and procedure, is not correct. As
stated in “Guidelines for Examination of Patent Applications under 35 U.S.C. 112, ¶ 1, ‘Written
Description’ Requirement,” 66 F.R. 1099, 1106 (Jan. 5, 2001), right column, “[W]hen there is
substantial variation within a genus, an applicant must describe a sufficient variety of species to
reflect the variation within the genus”; MPEP § 2163, paragraph III. A. 3. (a)(ii) (pg. 2100-164)
(8th Ed.).
- Able and Baker conceived an improved gas grille for cookouts. Using elements A, B, C,
D, E and F found in their backyards, as well as elements G, H, I, J, K, L, M and N purchased at a
local hardware store, they successfully constructed and used a gas grille conforming to their
concept. The grille includes subcombination of elements K, L and M conceived by Able, and
subcombination C, D, F, G and M conceived by Baker. Able and Baker conceived their
respective subcombinations separately and at different times. Able and Baker retain you to
prepare and file a patent application for them. You are considering whether and what can be
claimed in one patent application. Which of the following is true?
(A) For Able and Baker to properly execute an oath or declaration under 37 CFR 1.63
in a patent application claiming not only the grille, but also the two
subcombinations, Able and Baker must be joint inventors of the grille, and each
of the two subcombinations.
(B) A characteristic of U.S. patent law that is generally shared by other countries is
that the applicant for a Changed by AIA patent must be the inventor.
(C) If Able and Baker execute an oath or declaration under 37 CFR 1.63 as joint
inventors and file an application claiming the grille (a combination of elements A,
B, C, D, E, F, G, H, I, J, K, L, M and N), the existence of the claim to the grille is
evidence of their joint inventorship of the individual elements.
(D) Able and Baker may properly execute an oath or declaration under 37 CFR 1.63
as joint inventors and file an application containing claims to the grille (a
combination of elements A, B, C, D, E, F, G, H, I, J, K, L, M and N), claims to
the subcombination conceived by Able, and claims to the subcombination
conceived by Baker.
(E) Able and Baker could not properly claim the combination unless they successfully
and personally reduced the grille to practice.
§ 2137.01, “Inventorship,” and see Kimberly-Clark Corp. v. Procter & Gamble Distributing, 23
USPQ2d 1921, 1925 – 26 (Fed. Cir. 1992); and Moler v. Purdy, 131 USPQ 276, 279 (Bd. Pat.
Inter. 1960). (A) is not correct. MPEP § 2137.01 (Requirements for Joint Inventorship) and see
Kimberly-Clark Corp. v. Procter & Gamble Distributing, 23 USPQ2d 1921, 1925 – 26 (Fed. Cir.
1992); and Moler v. Purdy, 131 USPQ 276, 279 (Bd. Pat. Inter. 1960). (B) is not correct. 35
U.S.C. §§ 101, 115; MPEP § 2137.01. (C) is not correct. MPEP § 2137.01. The inventor of an
element, per se, and the inventor of a combination using that element may differ. See In re
DeBaun, 214 USPQ 933, 936 (CCPA 1982); and In re Facius, 161 USPQ 294, 301 (CCPA
1969). (E) is not correct. There is no provision in the Patent Statute requiring the invention to
be reduced to practice in order to file a patent application claiming the invention. Further, see
MPEP § 2137.01; and see In re DeBaun, 214 USPQ 933, 936 (CCPA 1982).
**26. Where a reference relied upon in a 35 U.S.C. § 103 rejection qualifies as prior art only
under 35 U.S.C. § 102(f), or (g), which of the following represents the most comprehensive
answer in accord with proper USPTO practice and procedure as to the action an applicant should
take to overcome the rejection?
(A) Present proof that the subject matter relied upon and the claimed invention are
currently commonly owned.
(B) Present proof that the subject matter relied upon and the claimed invention were
commonly owned at the time the later invention was made.
(C) Present proof that the subject matter relied upon and the claimed invention were
subject to an obligation to assign to the same person at the time the later invention
was made.
(D) (A) and (B).
(E) (C) and (D).
- All answers accepted.
- Which of the following is most likely to be considered in a proper obviousness
determination?
(A) Evidence demonstrating the manner in which the invention was made.
(B) Evidence that a combination of prior art teachings, although technically
compatible, would not be made by businessmen for economic reasons.
(C) Evidence demonstrating the level of ordinary skill in the art.
(D) Evidence that one of ordinary skill in the art, after reading Kat’s application,
would readily be able to make and use Kat’s invention without undue
experimentation.
(E) Evidence that the distance finder described in the July 2000 golf magazine has
enjoyed great commercial success.
- ANSWER: The most correct answer is (C). The level of ordinary skill in the art is one of
the factors that must be considered in any obviousness determination. Graham v. John Deere,
383 U.S. 1, 148 USPQ 459 (1966). (A) is not the best answer because 35 U.S.C. § 103
specifically states that patentability shall not be negated by the manner in which the invention
was made. (B) is not the best answer because economic unfeasibility is not a basis for a
determination of nonobviousness. See MPEP § 2145 VII. (D) is directed to the issue of
enablement, not obviousness. (E) is wrong because the commercial success of the prior art
distance finder is not relevant (although commercial success of Kat’s invention would be
relevant).
- A U.S. patent application discloses a first embodiment of an invention, a composition
made of known materials in equal amounts by weight of A, B, and C. The application discloses
a second embodiment of the invention comprising equal amounts by weight of A, B, and C, and
an effective amount of D, a known material, to reduce excess moisture from the composition.
The application also discloses a third embodiment of the invention comprising equal amounts by
weight of A, B, and C, and an effective amount of D to reduce the acidity of the composition.
The application fully discloses guidelines for determining an effective amount of D to reduce
excess moisture from the composition, and determining an effective amount of D to reduce the
acidity of the composition. The application discloses that the amount of D needed to reduce
excess moisture from the composition differs from the amount of D needed to reduce the acidity
of the composition. Which of the following claims, if included in the application, provides a
proper basis for a rejection under 35 U.S.C. § 112, second paragraph?
(A) Claim 1. A composition comprising: equal amounts by weight of A, B, and C, and
an effective amount of D to reduce the acidity of the composition.
(B) Claim 1. A composition comprising: equal amounts by weight of A, B, and C, and
an effective amount of D.
(C) Claim 1. A composition comprising: equal amounts by weight of A, B, and C, and
an effective amount of D to reduce excess moisture from the composition.
(D) Claim 1. A composition comprising: equal amounts by weight of A, B, and C.
(E) None of the above.
- ANSWER: (B) is the correct answer. 35 U.S.C. § 112, second paragraph and MPEP
§ 2173.05(c)(III). The claim presented in (B) is improper as “an effective amount” has been held
to be indefinite when the claim fails to state the function that is to be achieved and more than one
effect can be implied from the specification. In re Fredericksen 213 F.2d 547, 102 USPQ 35
(CCPA 1954). It is unclear whether “an effective amount” in (B) is an effective amount to
reduce acidity or an effective amount to reduce moisture. The claims presented in (A) and (C)
find support in the disclosure, which provides guidelines for determining “an effective amount”
for each of the claims in (A) and (C). MPEP § 2173.05(c)(III). The claim presented in (D) is not
indefinite, given that A, B, and C are known materials as set forth in the question and the
composition can be determined by the claim language. (E) is incorrect because (B) is correct.
- Practitioner Smith filed a utility patent application on January 5, 2001, with informal
drawings. Upon review of the drawings, the USPTO concluded that the drawings were not in
compliance with the 37 CFR 1.84(a)(1) and (k), and were not suitable for reproduction. In an
Office communication, Smith was notified of the objection and given two months to correct the
drawings so that the application can be forwarded to a Technology Center for examination.
Which of the following complies with USPTO practices and procedures for a complete bona fide
attempt to advance the application to final action?
(A) Smith timely files a response requesting that the objections to the drawings be
held in abeyance until allowable subject matter is indicated.
(B) Smith timely files a response requesting that the objections to the drawings be
held in abeyance since the requirement increases up- front costs for the patent
applicant, and the costs can be avoided if patentable subject matter is not found.
(C) Smith timely files a response requesting that the objections to the drawings be
held in abeyance until fourteen months from the earliest claimed priority date.
(D) Smith timely files a response correcting the drawings to comply with 37
CFR 1.84(a)(1) and (k), and making them suitable for reproduction.
(E) All of the above.
- ANSWER: (D). Under 37 CFR 1.85(a), correcting the drawings to comply with 37 CFR
1.84(a)(1) and (k), and making them suitable for reproduction is a bona fide response. (A), (B),
and (C) are not the most correct answer. In each, Smith seeks to hold the requirement in
abeyance. As stated in 37 CFR 1.85(a) (effective November 29, 2000), “Unless applicant is
otherwise notified in an Office action, objections to the drawings in a utility or plant application
will not be held in abeyance, and a request to hold objections to the drawings in abeyance will
not be considered a bona fide attempt to advance the application to final action.” See also,
“Changes to Implement Eighteen-Month Publication of Patent Applications; Final Rule,” 65 F.R.
57024, 57032, “Section 1.85.” (E) is not the most correct answer inasmuch as (A), (B), and (C)
are not the most correct answers.
- According to USPTO rules and procedure, which of the following can be overcome by an
affidavit under 37 CFR 1.131?
(A) A rejection properly based on statutory double patenting.
(B) A rejection properly made under 35 U.S.C. § 102(d) based on a foreign patent
granted in a non-WTO country.
(C) A rejection properly made under 35 U.S.C. § 102(a) based on a journal article
dated one month prior to the effective filing date of the U.S. patent application.
Applicant has clearly admitted on the record during the prosecution of the
application that subject matter in the journal article relied on by the examiner is
prior art.
(D) A rejection properly made under 35 U.S.C. § 102(b) based on a U.S. patent that
issued 18 months before the effective filing date of the application. The patent
discloses, but does not claim, the invention.
(E) None of the above.
- ANSWER: (E) is the correct answer. MPEP § 715. (A) is incorrect because an affidavit
under 37 CFR 1.131 is not appropriate where the reference is a prior U.S. patent to the same
entity, claiming the same invention. MPEP § 715. (B) and (D) are each incorrect because an
affidavit under 37 CFR 1.131 is not appropriate where the reference is a statutory bar under 35
U.S.C. § 102(d) as in (B) or a statutory bar under 35 U.S.C. § 102(b) as in (D). MPEP § 715.
(C) is incorrect because an affidavit under 37 CFR 1.131 is not appropriate where applicant has
clearly admitted on the record that subject matter relied on in the reference is prior art. MPEP
§ 715.
Questions 11 and 12 are based on the following factual background. Consider questions 11 and 12 independently of each other.
Applicant files a patent application in Japan on February 28, 1996. Applicant files a PCT international application designating the United States on February 27, 1997, based on the Japanese application. The international application is published in English on August 28, 1997. The international application enters the national stage in the United States on August 28, 1998. The USPTO publishes the application on June 7, 2001 at the request of the applicant. The application issues as a United States patent on December 4, 2001.
11. When examining an application filed on or after November 29, 2000 or any application that has been voluntarily published, what is its earliest possible prior art date, for the June 7th U.S. published application in view of 35 U.S.C. § 102(e) as amended by the American Inventors Protection Act of 1999? (A) February 28, 1996. (B) February 27, 1997. (C) August 28, 1997. (D) August 28, 1998. (E) June 7, 2001.
- ANSWER: (B) is the most correct answer. 35 U.S.C. § 102(e)(1) provides that a USPTO published application, based on an earlier international application, has prior art effect as of its international filing date, if the international application designated the United States, and was published in English. Because in the above fact pattern, the international application designated the United States and was published in English, the USPTO published application is entitled to its international filing date of February 27, 1997 for prior art purposes. (A) is wrong because the Japanese filing date is relevant under 35 U.S.C. § 119 only for priority, but not for prior art purposes. (C) and (E) are wrong because they recite prior art dates that are later than February 27, 1997. (D) is wrong because the amendments to § 102(e) by the American Inventors Protection Act of 1999 make the national stage entry date irrelevant for prior art purposes.
**16. Jill, a registered patent agent, receives a Notice of Allowance from the USPTO
with a mail date of November 13, 2001, regarding a utility patent application for an
improved garden hose which she had filed on behalf of one of her small entity clients.
The Notice of Allowance specifies a sum that must be paid within three months of the
mailing date to avoid abandonment. The sum specified includes both the issue fee and
the publication fee. As a result of a small fire in her office building, Jill is unable to
resurrect her files until the last day of the three month period. Jill mails a letter to the
USPTO on February 13, 2002 using the U.S. Postal Service. Jill does not employ the
procedures of 37 CFR 1.8 or 1.10 to mail the letter. The letter is received in the USPTO
on February 15, 2002. The letter correctly identifies the application. The letter
authorizes the USPTO to charge the proper issue fee for a small entity to her deposit
account. The account has been identified in a previously filed authorization to charge
fees. At the time the letter was filed in the USPTO, the account had a balance of
$1000.00 in funds. Nothing in the letter authorized payment of the publication fee, no
petition for an extension of time was filed, and an Office-provided issue fee transmittal
form was not filed. No postal emergency was involved in filing the letter. Which of the
following statements accords with proper USPTO practice and procedure?
(A) The application will become abandoned because Jill did not authorize
payment of the publication fee.
(B) The application will not become abandoned because the authorization to
charge fees operates as a request to charge the correct fees to any deposit
account identified in a previously filed authorization to charge fees.
(C) The application will become abandoned because Jill’s letter did not
include a petition for an extension of time accompanied by the proper fee.
(D) The application will become abandoned because a completed Officeprovided
issue fee transmittal form, PTOL-85B, did not accompany Jill’s
letter.
(E) The application will become abandoned because Jill’s letter was not
timely filed in the USPTO and it was not mailed in accordance with the
provisions of 37 CFR 1.8 or 1.10.
- ANSWER: (E) is correct. A communication mailed within the time given for response in
accordance with the procedure of 37 C.F.R. § 1.8 or 1.10 is considered timely filed even if it is
received after the date a reply is due. In (E), the letter was not mail in accordance with 37
C.F.R.§ 1.8 or 1.10. Therefore, the letter would be considered filed when it is received in the
USPTO. 37 C.F.R. § 1.311(a) states, “This three month period is not extendable.” Without
complying with 37 C.F.R. § 1.8 or 1.10, the filing date of the letter is the date it was received in
the USPTO, i.e., February 15, 2002, which is after the due date for payment of the issue fee,
February 13, 2002. (A) is wrong because the reason given for abandonment is incorrect. The
application becomes abandoned for the reasons expressed in (E). Although 37 C.F.R. § 1.311(b)
provides that the submission of an incorrect issue fee (or other post-allowance fees set forth in 37
C.F.R. § 1.18) operates as a request to charge the correct issue fee, it does not change the fact
that Jill’s letter is received by the USPTO after expiration of the non-extendable statutory three
month period for payment of the issue fee. (B) is wrong because the application will become
abandoned because Jill’s letter will be received by the USPTO after expiration of the nonApril
extendable statutory three month period. (C) is wrong because the period for payment of the
issue fee was not extendable by petition. The period is set by statute. 35 U.S.C. § 151.
Abandonment occurred because Jill’s letter was received by the USPTO after expiration of the
non-extendable statutory three month period. A petition for an extension of time was not
available in this case. (D) is wrong because the reason for abandonment is wrong. Filing the
form is optional. Although 37 C.F.R. § 1.311(b) provides that the submission of a completed
Office-provided issue fee transmittal form, PTOL-85B, operates as a request to charge the
correct issue fee (or other post-allowance fees set forth in 37 C.F.R. § 1.18) to any deposit
account identified in a previously filed authorization to charge fees, it does not change the fact
that Jill’s letter was received by the USPTO after expiration of the non-extendable statutory three
month period.
- Mr. Brick, the inventor, files an application with the USPTO on January 2, 2001
containing a single claim for his invention: a new bouncing ball called “Y”. Brick
receives a first Office action dated June 4, 2001 from the primary examiner handling
Brick’s application. The examiner rejected Brick’s claim only under 35 U.S.C. § 103 on
the grounds that Reference X teaches a bounc ing ball called “Q,” and that although “Y”
and “Q” are not the same, it would have been obvious to one of ordinary skill to make
changes to the “Q” ball in order to obtain a ball just like Brick’s “Y” ball.
On August 2, 2001, Brick responds by stating that his new “Y” ball bounces
unexpectedly higher than the “Q” ball described in Reference X. Brick includes a
declaration, signed by Mrs. Kane, that includes extensive data comparing the bouncing
results for the “Y” and “Q” balls and showing that the “Y” ball bounces unexpectedly
higher than the “Q” ball. Brick argues that the rejection under 35 U.S.C. § 103 should be
withdrawn because he has proven that, in view of the unexpectedly higher bounce of the
“Y” ball as compared to the “Q” ball, it would not have been obvious to one of ordinary
skill in the art to make changes to the “Q” ball to obtain Brick’s “Y” ball.
On October 2, 2001, Brick receives a final rejection from the examiner. The rejection
states, in its entirety: “The response has been reviewed but has not been found persuasive
as to error in the rejection. The claim is finally rejected under 35 U.S.C. § 103 for the
reasons given in the first Office action.” Brick believes he is entitled to a patent to his
new bouncing ball “Y.” How should Brick proceed?
(A) Brick should give up because the declaration did not persuade the
examiner of the merits of Brick’s invention.
(B) Brick should timely file a Request for Reconsideration asking the
examiner to reconsider the rejection on the basis of the Kane declaration
and, as a precaution against the Request for Reconsideration being
unsuccessful, also timely file a Notice of Appeal.
(C) Brick should respond by submitting a request for reconsideration
presenting an argument that Reference X does no t provide an enabling
disclosure for a new ball with the unexpectedly higher bounce of his “Y”
ball.
(D) Brick should respond by submitting a request for reconsideration
presenting an argument that Reference X does not provide a written
description for a new ball with the unexpectedly higher bounce of his “Y”
ball.
(E) Brick should respond by submitting a request for reconsideration
presenting an argument the declaration data proves that the “Q” ball and
the “Y” are not identical.
- ANSWER: (B) is the correct answer. It is inappropriate and injudicious to disregard any
admissible evidence in any judicial proceeding. Stratoflex, Inc. v. Aeroquip Corp., 713 F.2d
1530, 218 USPQ 871 (Fed. Cir. 1983). The examiner has not analyzed the data in the declaration
nor provided an explanation as to why the declaration did not overcome the rejection.
Furthermore, the rejection has not been reviewed anew in light of the declaration. The examiner
should have reweighed the entire merits of the prima facie case of obviousness in light of the
data. In re Hedges, 783 F.2d 1038, 1039, 228 USPQ 685, 686 (Fed. Cir. 1986). Accordingly,
Block should ask that the rejection be reconsidered and file a Notice of Appeal to safeguard his
interest for a review of the rejection by the Board of Patent Appeals and Interferences if the
rejection is not reconsidered. 37 C.F.R. § 1.116. (A) is wrong because there is no evidence that
the examiner made any review of the declaration. (C) is wrong because whether or not Reference
X provides an enabling disclosure for Block’s invention is immaterial to the question of
obviousness. If there were to be a question of enabling disclosure for Reference X, it would be
with respect to the “Q” ball relied upon by the examiner, not applicant’s “Y” ball. (D) is wrong
because whether or not Reference X provides a written description for Block’s invention is
immaterial to the question of obviousness raised by the examiner. (E) is wrong because the issue
is one of obviousness under 35 U.S.C. § 103, not identity under 35 U.S.C. § 102. Given that the
examiner has rejected the claim under 35 U.S.C. § 103 and not under § 102, the examiner has
already conceded that the “Q” and “Y” balls are not the same.
- Johnnie owns a supermarket store in Cleveland, Ohio, and is constantly frustrated
when little children drop their chewing gum on Johnnie’s clean floor in the supermarket.
In her spare time, Johnnie develops an entirely novel type of coating material that she
applies to floor tile. The coating material resists adhesion to chewing gum. In order to
check out the effectiveness of the floor tile coating material, on December 31, 2000, she
secretly covers the floor tiles in her supermarket with the new chewing gum resistant
floor tile coating material. Johnnie is amazed at the results inasmuch as cleaning the
floor was never easier. On January 30, 2001, Johnnie, satisfied with the experimental use
results, ceased testing the use of the coating material. The ability of the coating material
to withstand chewing gum adhesion continued unabated throughout the remainder of - On January 1, 2002, one of Johnnie’s many customers, James, remarked at how
clean the floor looked. Johnnie then told James of her invention. James thinks for one
moment and suggests that the floor tile coating material may be useful in microwave
ovens, so that food will not stick to the interior sides of the microwave oven. James
discusses getting patent protection with Johnnie. Which of the following is true?
(A) Johnnie could never be entitled to a patent on a floor tile in combination
with a coating material affixed to the outer surface of the tile.
(B) James can be named as a coinventor with Johnnie in a patent application
claiming a microwave oven wherein the internal surfaces of the oven are
coated with the coating material.
(C) Since for one year Johnnie told nobody that the floor tile in her
supermarket contained the new chewing gum resistant coating material,
she would never be barred from obtaining patent protection for the floor
coating material.
(D) Use of the floor tile coating material in microwave ovens would have been
obvious to one of ordinary skill in the art, since James thought of it within
seconds after first learning of the floor tile coating material, and James
was not skilled in the art.
(E) The floor tile having the coating material affixed to the outer surface of
the tile, an article of manufacture, would not be patentable as of January 1,
2002 inasmuch as the article was in public use on the supermarket floor
for one year.
- ANSWER: (B). Since Johnnie developed the material and James thought of the idea to use
it in microwave ovens, they rightfully could be considered coinventors of the new article of
manufacture. As to (A) and (C), public use began on when the experimental use ended on
January 30, 2001, and occurs even when the public is unaware that they were walking on the
developed material since the material was used in a public place. As to (D), even though James
only took a second to think of the idea, he is entitled to receive a patent unless it was obvious to
one of ordinary skill in the art. Nothing in the prior art revealed that it was obvious to use the
material in microwave ovens. As to (E), the article of manufacture is not barred even though the
floor material itself cannot be patented. Johnnie conducted an experimental use of the article
from December 31, 2000 through January 30, 2001. Thereafter, Johnnie had one year from the
end date of the experimental use to file a patent application for the article. Johnnie may file a
patent application before January 30, 2002.
**50. Which of the following is not a USPTO recommendation or requirement?
(A) Claims should be arranged in order of scope so that the first claim presented is the least restrictive.
(B) Product and process claims should be separately grouped.
(C) Every application should contain no more than three dependent claims.
(D) A claim which depends from a dependent claim should not be separated from that dependent claim by any claim which does not also depend from the dependent claim.
(E) Each claim should start with a capital letter and end with a period.
- ANSWER: (C). The USPTO does not require or recommend a minimum or maximum number of dependent claims. 37 C.F.R. § 1.75(c). (A) is a USPTO recommendation. See MPEP 608.01(m) (“Claims should preferably be arranged in order of scope so that the first claim presented is the least restrictive.”). (B) is a USPTO recommendation. See MPEP 608.01(m) (“Similarly, product and process claims should be separately grouped.”). (D) is a PTO recommendation. See MPEP 608.01(n), part IV. (E) is a USPTO requirement. See MPEP 608.01(m) (“Each claim begins with a capital letter and ends with a period.”).
- An amendment filed in January 8, 2002, in an unassigned nonprovisional
application seeks to cancel claims so that fewer than all of the currently named inventors
are the actual inventors of the invention being claimed. The amendment includes a
request to delete the names of the persons who are not inventors. In accordance with
proper USPTO rules and procedure, the request may be signed by which of the
following?
(A) A registered practitioner not of record who acts in a representative
capacity under 37 CFR 1.34(a).
(B) All of the applicants (37 CFR 1.41(b)) for patent.
(C) A registered practitioner of record appointed pursuant to 37 CFR 1.34(b).
(D) (B) and (C).
(E) (A), (B), and (C).
- ANSWER: (E) is the correct answer. 37 C.F.R. § 1.48(b) (effective November 7, 2000);
“Changes To Implement the Patent Business Goals; Final Rule,” 65 FR 54604, 54619
(September 8, 2000). As stated in 65 FR at 54619, middle column, “Sections 1.48(b) and (d) are
revised to indicate that a request to correct the inventorship thereunder must be signed by a party
as set forth in § 1.33(b)…” (A), (B), and (C) are provided for in 37 C.F.R. § 1.33(b). Thus (E),
the most inclusive answer, is correct.
- Which of the following is not required in order for a foreign application that has
matured into a foreign patent to qualify as a reference under 35 U.S.C. § 102(d)?
(A) The foreign application must have actually been published before the
filing of an application in the United States, but the patent rights granted
need not be enforceable.
(B) The foreign application must be filed more than 12 months before the
effective filing date of the United States application.
(C) The foreign and United States applications must be filed by the same
applicant, his or her legal representatives or assigns.
(D) The foreign application must have actually issued as a patent or inventor’s
certificate before the filing of an application in the United States. It need
not be published but the patent rights granted must be enforceable.
(E) The same invention must be involved.
- ANSWER: (A) is the most correct answer. 35 U.S.C. § 102(d). The foreign application
need not be published, but the patent rights granted must be enforceable. MPEP § 706.02(e).
(B), (C), (D) and (E) are required by 35 U.S.C. § 102(d).
- For purposes of determining whether a request for continued examination is in
accordance with proper USPTO rules and procedure, in which of the following situations
will prosecution be considered closed?
(A) The last Office action is a final rejection.
(B) The last Office action is an Office action under Ex Parte Quayle.
(C) A notice of allowance has issued following a reply to a first Office action.
(D) The application is under appeal.
(E) All of the above.
All of the answers relate to the evidence of the close of prosecution and, hence, E
is the answer since it includes all of them.
- Assume that conception of applicant’s complex invention occurred prior to the
date of the reference, but reduction to practice occurred after the date of the reference.
Which of the following is sufficient to overcome the reference in accordance with proper
USPTO practice and procedure?
(A) In a 37 CFR 1.131 affidavit or declaration, it is sufficient to allege that
applicant or patent owner has been diligent.
(B) In a 37 CFR 1.131 affidavit or declaration, it is sufficient to clearly
establish conception of the invention prior to the effective date of the
reference, and diligence from just prior to the effective date of the
reference to actual reduction to practice. The presence of a lapse of time
between the reduction to practice of an invention and the filing of an
application thereon is not relevant.
(C) In a 37 CFR 1.131 affidavit or declaration, it is sufficient to clearly
establish conception of the invention prior to the effective date of the
reference. Diligence need not be considered.
(D) In a 37 CFR 1.131 affidavit or declaration, it is sufficient to show
conception and reduction to practice in any country.
(E) In a 37 CFR 1.131 affidavit or declaration, it is always sufficient to prove
actual reduction to practice for all mechanical inventions by showing plans
for the construction of the claimed apparatus.
(B) is the most correct answer. See Ex parte Merz, 75 USPQ 296 (Bd. App.
1947) (holding that the “lapse of time between the completion or reduction to practice of an invention and the filing of an application thereon” is not relevant to an affidavit or declaration under 37 C.F.R. § 1.131(b)); MPEP § 715.07(a). (A) is incorrect. Ex parte Hunter, 1889 C.D. 218, 49 O.G. 733 (Comm’r Pat. 1889); MPEP § 715.07(a). Applicant must show evidence of facts establishing diligence. (C) is incorrect. Ex parte Kantor, 177 USPQ 455 (Bd. App. 1958)
(after conception has been clearly established, diligence must be considered prior to the effective date is clearly established, since diligence then comes into question); MPEP § 715.07(a). (D) is incorrect. MPEP § 715.07(c). 37 C.F.R. § 1.131(a) provides for the establishment of a date of completion of the invention in a NAFTA or WTO member country, as well as in the United States, an applicant can establish a date of completion in a NAFTA member country on or after December 8, 1993, the effective date of section 331 of Public Law 103 – 182, the North American Free Trade Agreement Act, and can establish a date of completion in a WTO member country other than a NAFTA member country on or after January 1, 1996, the effective date of section 531 of Public Law 103 – 465, the Uruguay Round Agreements Act. Not all countries are members of NAFTA or WTO, and prior invention in a foreign country cannot be shown without regard for when the reduction to practice occurred. (E) is incorrect. MPEP § 715.07. Actual reduction to practice generally, but not always, requires a showing that the apparatus actually
existed and worked, “There are some devices so simple that a mere construction of them is all that is necessary to constitute reduction to practice.” In re Asahi/America Inc., 68 F.3d 442, 37 USPQ2d 1204 (Fed. Cir. 1995) (citing Newkirk v. Lulegian, 825 F.2d 1581, 3USPQ2d 1793 (Fed. Cir. 1987) and Sachs v. Wadsworth, 48 F.2d 928, 929, 9 USPQ 252, 253 (CCPA 1931).
The claimed restraint coupling held to be so simple a device that mere construction of it was sufficient to constitute reduction to practice. Photographs, coupled with articles and a technical report describing the coupling in detail were sufficient to show reduction to practice.).
While vacationing in Mexico on April 14, 2001, Henrietta invented a camera that
operated at high temperature and is waterproof. She carefully documented her invention
and filed a provisional application in the USPTO on April 30, 2001. She conducted tests
in which the camera withstood temperatures of up to 350 degrees Fahrenheit. However,
when the camera was placed in the water leaks were discovered rendering the camera
inoperable. On April 12, 2002, Henrietta conceived of means that she rightfully believed
will fix the leakage issue. Henrietta came to you and asked whether she can file another
application. Henrietta desires to obtain the broadest patent protection available to her.
Which of the following is the best manner in accordance with proper USPTO practice
and procedure for obtaining the patent covering both aspects of her invention?
(A) She can file a nonprovisional application on April 30, 2002 claiming
benefit of the filing date of the provisional application, disclosing the
means for fixing the leak and presenting a claim covering a camera that
operates at high temperatures and a claim covering a camera that is
waterproof, or presenting a claim covering a camera that both operates at
high temperatures and is waterproof.
(B) Henrietta cannot rightfully claim a camera that is waterproof in a
nonprovisional application filed on April 30, 2002, since she tested the
camera and the camera developed leaks.
(C) Henrietta can file another provisional application on April 30, 2002 and
obtain benefit of the filing of the provisional application filed on April 30,
2001.
(D) Henrietta may establish a date of April 14, 2001 for a reduction to practice
of her invention for claims directed to the waterproofing feature.
(E) Henrietta should file a nonprovisional application on April 30, 2002
having claims directed only to a camera that withstands high temperatures
since the camera that she tested developed leaks.
A provisional holds your place in line for whatever it includes in its disclosure. If
you add more stuff when the non-provisional is filed you can still obtain benefit
for whatever the two applications shared. A is the answer.
ANSWER: (A). As to (B) and (E), an actual reduction to practice is not a necessary
requirement for filing an application so long as the specification enables one of ordinary skill in
the art to make and use the invention. However, (D) is incorrect, as a reduction to practice may
not be established since the camera leaked. As to (C), a second provisional is not entitled to the
benefit of the filing date of the first provisional application. 35 U.S.C. § 111(h)(7).
Roger Rocket is a designer of paper cups at Paper America. During his free time, he
likes to attend baseball games at Yankee Stadium. One day, while seated in the stands,
he caught a fly ball. He took the baseball home and played catch with his friends Andy
Cannon, Orlando Torpedo, and Mariano Missle. Unfortunately for Rocket, Cannon has a
problem with accuracy. Cannon threw the ball over Rocket’s head and straight through a
neighbor’s front window. The shattered glass ripped the lining off of the baseball.
Instantly, Rocket conceived a more durable baseball with an exterior similar to that of a
golf ball. Rocket worked for months on his invention in Missle’s garage. His new
baseball was comprised of a titanium core, and a plastic shell having circular dimples and
V-shaped laces. Torpedo realized and told Rocket that Y-shaped laces would enable
baseball players to throw the ball faster. Cannon, an engineer in a radar gun laboratory,
tested the velocity of the baseball with both V and Y-shaped laces. To Cannon’s surprise,
the baseball traveled 10 M.P.H. faster with the Y-shaped laces. Rocket wanted patent
protection for a baseball having a titanium core, and a plastic shell having circular
dimples and Y-shaped laces, so he approached Yogi Practitioner for assistance. Rocket
has no obligation, contractual or otherwise, to assign his inventions to Paper America.
In accordance with proper USPTO practice and procedure, who should execute
the oath?
(A) Rocket (B) Rocket and Torpedo (C) Rocket and Cannon (D) Rocket, Torpedo, and Cannon (E) Rocket, Torpedo, Cannon, and Missle
Before executing the oath, Rocket wanted to ask Practitioner a question. On his
way to Practitioner’s office, Rocket was instantly killed when a drunk driver hit his car.
The officers or employees of Paper America are not related to Rocket. Who can execute
an oath on Rocket’s behalf?
(A) The President of Paper America (B) The CEO of Paper America (C) Rocket’s manager at Paper America (D) Rocket’s legal representative (E) None of the above
Practitioner received all of the proper papers required to
receive a filing date. However, due to an unexpected emergency, he had to fly out of the
country that evening to conduct discovery in another matter. Practitioner knew that he
would be out of the office for at least 4 weeks, so before leaving, he left a note instructing
his assistant to file the Rocket application on October 13, 2001, using an Express Mailing
label. His assistant did not see the note until 8:00 P.M. on Friday, October 19, 2001. On
Monday, October 22, 2001, Rocket’s assistant deposited the Rocket application in the
United States Postal Service with a proper Express Mailing label. The Postal Service
properly completed a legible label showing an October 22, 2001 date in. The
correspondence was received in the USPTO on October 27, 2001. What is the filing date
of the Rocket application absent any Postal Service Emergency?
(A) October 12, 2001 (B) October 13, 2001 (C) October 19, 2001 (D) October 22, 2001 (E) October 27, 2001
- Joint inventorship is based on conception only, not reduction to practice. Rocket
conceived of the basic titanium ball and Torpedo came up with the laces. B is the
answer. - Bizarre macabre facts. A PTO specialty. An estate rep. can sign for Rocket since
Paper America is completely out of the picture. - Express Mail “date in” is the filing date. Answer D.
- Patentee, Iam Smarter, filed and prosecuted his own nonprovisional patent
application on November 29, 1999, and received a patent for his novel cellular phone on
June 5, 2001. He was very eager to market his invention and spent the summer meeting
with potential licensees of his cellular phone patent. Throughout the summer of 2001, all
of the potential licensees expressed concern that the claim coverage that Smarter obtained
in his cellular phone patent was not broad enough to corner the market on this
technology, and therefore indicated to him that they feel it was not lucrative enough to
meet their financial aspirations. By the end of the summer, Smarter is discouraged. On
September 5, 2001, Smarter consults with you to find out if there is anything he can do at
this point to improve his ability to market his invention. At your consultation with
Smarter, you learn the foregoing, and that in his original patent application, Smarter had a
number of claims that were subjected to a restriction requirement, but were nonelected
and withdrawn from further consideration. You also learn that Smarter has no currently
pending application, that the specification discloses Smart’s invention more broadly than
he ever claimed, and that the claims, in fact, are narrower than the supporting disclosure
in the specification. Which of the following will be the best recommendation in
accordance with proper USTPO practice and procedure?
(A) Smarter should immediately file a divisional application under 37 CFR
1.53(b) including the nonelected claims that were subjected to a restriction
requirement in the nonprovisional application that issued as the patent.
(B) Smarter should file a reissue application under 35 U.S.C. § 251, including
the nonelected claims that were subjected to the restriction requirement in
the nonprovisional application that issued as the patent.
(C) Smarter should file a reissue application under 35 U.S.C. § 251,
broadening the scope of the claims of the issued patent, and then file a
divisional reissue application presenting only the nonelected claims that
were subjected to a restriction requirement in the nonprovisional
application which issued as the patent.
(D) Smarter should simultaneously file two separate reissue applications under
35 U.S.C. § 251, one including broadening amendments of the claims in
the original patent, and one including the nonelected claims that were
subjected to a restriction requirement in the nonprovisional application
which issued as the patent.
(E) Smarter should file a reissue application under 35 U.S.C. § 251 on or
before June 5, 2003, broadening the scope of the claims of the issued
patent.
ANSWER: (E) is the correct answer. 35 U.S.C. § 251. The reissue permits Smarter to broaden the claimed subject (A) is incorrect. There must be copendency between the divisional application and the original application. 35 U.S.C. § 120. (B) This is incorrect, as an applicant’s failure to timely file a divisional application while the original application is still pending is not considered to be an error correctable via reissue, In re Orita, 550 F.2d 1277, 1280, 193 USPQ 145, 148 (CCPA 1977). (C) This is incorrect, as an applicant’s failure to timely file a divisional application while the original application is still pending is not considered to be an error correctable via reissue, Id., including a divisional reissue application. MPEP § 1402. (D)
This is incorrect, as an applicant’s failure to timely file a divisional application while the original application is still pending is not considered to be an error correctable via reissue, Id.
*24. In 1995 Patent Agent filed a U.S. patent application containing five claims
(Application 1). All five claims are fully supported under 35 U.S.C. § 112 by the
disclosure of Application 1. In 2000, Patent Agent filed a U.S. patent application
(Application 2) that was a continuation- in-part of Application 1. Application 2 adds new
subject matter to the disclosure of Application 1, and ten additional claims. Of the fifteen
claims in Application 2, claims 1-5 are exactly the same as Application 1, claims 6-10 are
fully supported under 35 U.S.C. § 112 by the disclosure of Application 1, and claims 11-
15 are fully supported under 35 U.S.C. § 112 only by the newly added subject matter of
Application 2. The effective filing date for claims in Application 2 is:
(A) 1-15 is 2000.
(B) 1-15 is 1995.
(C) 1-10 is 1995.
(D) 11-15 is 2000.
(E) (C) and (D).
ANSWER (E) The characterizing feature of a CIP is multiple filing dates. This question again
illustrates that the PTO wants you to know this feature. C and D set forth that the
claims are split between the two applications. Easy question if you remember the
CIP axiom of multiple filing dates. (By the way it also has multiple 102(e) dates
as a reference, but we’ll save that for another day!)
*37. Applicant Einstein files a patent application on November 26, 1999, that claims a new type of football pads. Prosecution is conducted and the application issues as a patent to Einstein on April 3, 2001. A competitor, Weisman, who has been making and selling football pads since April of 1998, learns of Einstein’s patent when Einstein approaches him on May 3, 2001, with charges of infringement of the Einstein patent. Weisman makes an appointment to see you to find out what he can do about Einstein’s patent, since Weisman believes that he is the first inventor of the claimed subject matter. At your consultation on May 17, 2001, with Weisman, you discover that Weisman widely distributed printed publications containing a fully enabling disclosure of the invention and all claimed elements in the Einstein patent. Weisman used the printed publication for marketing his football pads in April of 1998. Weisman explains that he wishes to avoid litigation. Which of the following is a proper USTPO practice and procedure that is available to Weisman?
(A) Weisman should file a petition to correct inventorship under 37 CFR 1.324 in the patent, along with a statement by Weisman that such error arose without any deceptive intention on his part, requesting that a certificate of correction be issued for the patent under 35 U.S.C. § 256, naming the correct inventive entity, Weisman.
(B) Weisman should file a reissue application under 35 U.S.C. § 251, requesting correction of inventorship as an error in the patent that arose or occurred without deceptive intention, wherein such error is corrected by adding the inventor Weisman and deleting the inventor Einstein, as well as citing Joe Weisman’s April 1998 printed publication for the football pads as evidence that Weisman is the correct inventor.
(C) Weisman should file a prior art citation under 35 U.S.C. § 301, citing the sales in April 1998 of football pads, and explain the pertinency and manner of applying such sales to at least one claim of the Einstein patent.
(D) Weisman should file a request for ex parte reexamination of the Einstein patent under 35 U.S.C. § 302, citing the April 1998 printed publication of football pads in, and explain the pertinency and manner of applying such prior art to at least one claim of the Einstein patent.
(E) Weisman should file a request for inter partes reexamination of the Einstein patent under 35 U.S.C. § 311, citing public use of the football pads in April 1998, and explain the pertinency and manner of applying such prior use to at least one claim of the Einstein patent.
ANSWER: (D) is correct. It is the only answer that proposes to use a practice and procedure that is available to Einstein. 35 U.S.C. § 302. (A) This is incorrect because a statement by the currently named inventor as required by 37 C.F.R. § 1.324(b)(2) and the fee required by 37 C.F.R. § 1.20(b0 have not been filed. (B) This is incorrect, as in A.F. Stoddard & Co. v. Dann, 564 F.2d 556, 567 n.16, 195 USPQ 97, 106 n.16 (D.C. Cir. 1977) wherein correction of inventorship from sole inventor A to sole inventor B was permitted in a reissue application, does not apply here, as a reissue application can only be filed by the inventor(s) or assignee(s). See MPEP § 1412.04. (C) This answer is incorrect because it refers to sales, as opposed to patents or printed publications. (E) The option of requesting inter partes reexamination is not available in this scenario, as the patent in question issued from an original application which was filed prior to the critical date of November 29, 1999. Only patents which issued from original applications filed in the United States on or after November 29, 1999, are eligible for inter partes reexamination (37 C.F.R. § 1.913).
*38. Which of the following can correct the inventorship of a patent application in
accordance with proper USPTO practice and procedure?
(A) An unexecuted nonprovisional application was filed January 3, 2001
naming Jones and Smith as inventors. Smith was named an inventor in
error. A Notice to File Missing Parts of Application was mailed by the
Office, that requested a surcharge and an executed oath or declaration
under 37 CFR 1.63 by Jones and Smith. A registered practitioner in
timely response to the Notice submitted the requested surcharge and a
declaration under 37 CFR 1.63 that named only Jones as the inventor,
which declaration was only executed by Jones. The registered practitioner
had determined that a request to correct inventorship under 37 CFR
1.48(a) was unnecessary. No papers were submitted, by Smith, clarifying
that she is not an inventor.
(B) A nonprovisional application was filed January 3, 2001 with a declaration
under 37 CFR 1.63 naming Jones and Smith as inventors, which
declaration was signed only by Jones. Smith was named an inventor in
error. A Notice to File Missing Parts of Application was mailed by the
Office that requested a surcharge and an executed oath or declaration by
Smith. A registered practitioner timely responded to the Notice by
submitting the requested surcharge and a new declaration under 37 CFR
1.63 that identified Jones as the sole inventor, which declaration was
executed only by Jones.
(C) A nonprovisional application was filed February 28, 2000 that improperly
named Jones as the sole inventor in a declaration under 37 CFR 1.63.
Only Jones executed the declaration. Applicant need only re- file the
application as a continued prosecution application naming the correct
inventorship of Jones and Smith in the new application’s transmittal letter.
(D) A continuation application was filed under 37 CFR 1.53(b) using a copy
of an executed declaration from the prior application for which a
continuation was filed to correct the inventorship. The continuation
application papers were accompanied by a request by a registered
practitioner, in the continuation application transmittal paper, that Smith,
named as an inventor in the prior application, be deleted as an inventor in
the continuation application.
(E) (A) and (D).
- C is obviously wrong since it is a CPA and not a 53(b) filing. B is wrong since
the PTO will presume Jones signed on behalf of Smith, even though Smith’s
signature is missing. A and D are okay.